somme de vecteurs unitaires

Bonjour a tous,

voici un joli petit probleme qui ne semble pas si facile !
Si on choisit $n$ vecteurs unitaires de $\mathbb{R}^2$ aleatoirement uniformement sur le cercle unite
\[v_1,v_2, \ldots, v_n\]
est-il vrai que:
\[ \mathbb{P}(\|v_1+v_2+\ldots+v_n\| < 1 ) = \frac{1}{n+1}\]

des idees ?
«1

Réponses

  • Tu cherches vraiment une égalité?

    Sinon je vois deux pistes :

    1) Théorème central limite local (parce que c'est une marche aléatoire).

    2) Concentration de la mesure, à la Talagrand.
  • C'est quoi la norme ?
  • Bonjour Alekk.

    Le problème se retranscrit-il ainsi :
    Soit $(X_k)_k)$ une suite de variables aléatoire réelle indépendantes de même loi uniforme sur [0;1]. Prouver que $P(|\sum_{k=1}^n {e^{2 i \pi X_k}}| < 1) = \frac 1 {n+1}$ ?

    J'ai un doute sur le résultat, car pour n = 1 il est manifestement faux. C'est peut-être seulement pour n > 1 (Intuitivement, je trouve bien $\frac 1 3 $ pour n = 2).

    Cordialement.
  • oui, plusieurs precisions:
    1. la norme est la norme euclidienne usuelle: $\|(x,y)\|=\sqrt{x^2+y^2}$
    2. on cherche une egalite (resultats verifies par simulations) et non une inegalite
    3. $n \geq 2$, comme le fait remarquer Gerard0

    ps. le cas $n=2$ est facile, et on trouve bien $\frac{1}{3}$ (:P)
  • Salut à tous,

    En utilisant l'invariance par rotation des lois considérées, on voit que sauf erreur la suite des $r_n=||v_1+\cdots+v_n||^2$ est une chaîne de Markov, dont peut écrire une équation d'évolution sous la forme $r_{n+1}=(1+2 \alpha_{n+1})r_n+1$ où $\alpha_n$ est une suite i.i.d. égale en loi à l'abscisse de $v_n$.. ça permet peut-être de calculer par récurrence la fonction de répartition de $r_n$ (ou sa fonction caractéristique). Corrigez-moi si je dis des bêtises.
  • Heu, j'ai verifie pour n=3 et ca donne bien 1/4. Mais ce n'est deja pas si commode...D'ou vient ce probleme?
  • Erreur de calcul : c'est $r_{n+1}=1+r_n+2 \sqrt{r_n} \alpha_{n+1}$ bien sûr. Du coup la loi de $r_{n+1}$ conditionnellement à $r_n=s$ est une loi de type Arcsinus, portée par l'intervalle $\left[ \, (\sqrt{s}-1)^2 \, ; \, (\sqrt{s}+1)^2 \, \right]$.

    Bon mais ça ne mène pas très loin cette histoire.
  • Donc tu cherchais bien une égalité! Ouh là, si c'est vrai il existe forcément une très jolie preuve très astucieuse.
  • la loi de la somme a l'air d'avoir ete etudiee (cf: ici) et n'est pas bien belle. Cependant, si le resultat est vrai (et cela se verifie assez bien numeriquement), je pense qu'il doit y avoir un argument simple pour faire apparaitre ce $\frac{1}{n+1}$.

    On peut faire apparaitre ce $n$ si on ecrit:
    \[P(\|\sum_1^n v_i\|^2 < 1) = P( n + 2 \sum_{i < j} (v_i, v_j) < 1) = P(\sum_{i < j} (v_i, v_j) < \frac{1-n}{2})\]
    mais cela ne semble ne pas bien etre utile.
  • Merci alekk pour ce problème rigolo !
    De mon côté, j'essaie de passer par des Gaussiennes renormalisées comme modèle de tirage aléatoire (histoire de me ramener à des choses connues en quantités statistiques connues et étudiées, style théorème de Cochran et tutti quanti), mais ça ne me mène pas très loin pour le moment. Je ne désespère pas encore :)

    Amicalement,
  • J'ai une idée un peu dingo, inspirée d'un article de Kenyon & Winckler et qui s'appelle "Branched Polymers".

    Il s'agit de regarder ce qui se passe quand les longueurs des vecteurs sont respectivements $\epsilon^n,\epsilon^{n-1},\dots,\epsilon$. On pose $P_\epsilon$ la proba que pour ces longueurs on tombe dans la boule de centre $0$ et de rayon $\epsilon$. On est d'accord que l'on cherche $P_1$.

    Je pense que l'on peut calculer la dérivée de cette fonction, et intégrer.
  • Salut,
    Pour $n=1$ comme dit Gerard ca ne marche pas et meme la proba est nulle, elle
    vaudrait 1 si on remplace $<$ par $\leq$.

    Une idee a creuser peut-etre: on parametre les vecteurs par un angle, on note
    $K=[-\pi;\pi]$ La probabilité demandée s'ecrit:


    $$ P = \int_{K^n} \prod \frac{d\theta_i}{2\pi} I_{(\sum \cos(\theta_i) )^2 + (\sum \sin(\theta_i) )^2 < 1} $$
    avec $I$ la fonction caracteristique de $(\sum v_i)^2 < 1$.

    Ca revient a dire que si on considere un $v_0$ fixé,

    $$ P = \int_{K^{n}} \prod_{i=1}^n \frac{d\theta_i}{2\pi} I_{(v_0 + \sum_{i=1}^n v_i).(v_0 - \sum_{i=1}^n v_i)>0} $$
    et ca ne depend pas de la position de $v_0$ donc on peut moyenner dessus:

    $$ P = \int_{K^{n+1}} \prod_{i=0}^n \frac{d\theta_i}{2\pi} I_{(v_0 + \sum_{i=1}^n v_i).(v_0 - \sum_{i=1}^n v_i)>0} $$

    Notons $S = v_0 + \sum_{i=1}^n v_i = \sum_{i=0}^n v_i$ , la fonction
    caracteristique porte sur $S(2v_0 -S)>0$ c-a-d $2S.v_0 > S^2$
    Et j'aurais envie de dire que les domaines $2S.v_j > S^2$ forment un recouvrement de
    $K^{n+1}$ par $n+1$ domaine disjoints, sur lesquels par symetrie l'integrale aura
    la meme valeur. Mais ca, ca ne me parait pas evident a montrer (a supposer que ce soit vrai),
    mais en tout cas ca sonne bien pour conclure... ;-)

    A+

    Eric
  • Je ne vois toujours pas non plus. Mon intuition est également qu'il faut introduire une nouvelle variable de manière à trouver un vecteur $(W_0,\dots W_n)$ échangeable (iid par exemple) et une fonction $\phi$ telle que la proba cherchée puisse se réécrire $P(\forall i\in\{1,\dots,n\} \phi(W_0)<\phi(W_i))$, mais je ne vois pas bien comment la mettre en oeuvre.
  • n(x) = pr ( | somme des vi | <= x )

    x2 = y2 + 1 - 2y cos alpha

    pr (vi+1 dans secteur) = (1/pi) arcos ( (y2 - x2 + 1)/2y )

    pn+1 (x) = pn (max(0, x-1)) + intégrale entre max(0, x-1) et min(n, x+1) de (dy/pi) pn(y) arcos ( (y2 - x2 + 1)/2y )

    p2 (x) = (1/pi) arcos ( (2 - x2)/2 )

    idée à creuser ou débile ?14590
  • Bonjour,

    Voici une figure pour illustrer le rang 3
    A ce rang, après un choix aléatoire des deux permiers vecteurs, le retour dans la zone favorable reste (quasi) toujours possible.
    14631
  • Bonjour,

    A défaut d'avaoir su extirper quelque chose de mes gros calculs en Arccos, j'ai essayé de simuler les cas $n=4$ sur EXCEL
    par séries de 10 000 tests.

    14651
  • Numériquement, j'obtiens pour ma part des résultats compatibles avec alekk
    function r=simule(n,N)
      angle=rand(n,N,'u')*2*%pi
      lecos=cos(angle)
      lesin=sin(angle)
      r=sum((sum(lecos,'r').^2+sum(lesin,'r').^2)<=1)/N
    endfunction
    
    -->simule(4,100000)
     ans  =
     
        0.2003  
    
  • Arrêtez de chercher, j'ai la solution sous les yeux (je précise qu'elle n'est pas de moi...).

    J'essaie de vous taper ça d'ici ce soir (ça prend une petite page), pour info il y a une petite fonction de Bessel, un peu de Fourier.
  • J'attends avec impatience la solution !
    (sans vouloir te mettre la pression Lucas ;))

    Amicalement,
  • Eh oui,
    ma cellule G4 comportait une faute de frappe... que j'ai recopiée vers le bas!
    Là, pour 10 000 essais, j'ai 1968 succès, c'est plus proche du résultat théorique.
    Au moins, cette erreur aura-t-elle permis de faire remonter le topic.
    A toi Lucas de prendre le relais!
  • Il me semble qu'avec les données de Jacquot on obtient comme intervalle de confiance à $95\%$ pour la probabilité $p$ : $[0,189 \, ; \, 0.205]$ ce qui ne permet pas de rejeter l'hypothèse $p=0,2$ :)

    Sinon Lucas ta solution est effectivement alléchante, tu ne voudrais pas donner un petit hint pour que les motivés continuent à chercher ?
  • Moi, je dis, Lucas, il bluffe :)

    J'ai une super solution qui utilise le théorème de Feuerbach et le lemme de Nakayama, mais là, je peux pas la poster, j'ai piscine.
  • Ca s'appelle du marketing...

    La solution est de N.Enriquez (c'est mon chef) :

    Je note $X_i$ les pas, $S_n=\sum X_i$. On fait du Fourier : (j'utilise $\cdot$ pour le produit scalaire)
    \begin{align*}
    \mathbb{E}[e^{i\lamba S_n}]
    &= \mathbb{E}[e^{i\lamba \cdot X_1}]^n\\
    &=\left( \tfrac{1}{2\pi}\int_{[0,2\pi]} \exp(i\lambda_1 \cos\theta +i\lambda_2\sin\theta)d\theta \right)^n\\
    &= \left( \tfrac{1}{2\pi}\int_{[0,2\pi]} \exp(i|\lambda| \cos\theta )d\theta \right)^n\\
    &=: \phi(|\lambda|)^n,
    \end{align*}
    où $\phi(r)=\tfrac{1}{2\pi}\int\cos(r\cos\theta)d\theta$ (qui est la représentation de la fonction de Bessel $J_0$ sous forme intégrale).

    On inverse Fourier pour avoir la densité : calculons
    \begin{align*}
    \Psi(\mu)
    :&=\tfrac{1}{2\pi}\int_{\mathbb{R}^2} \exp(i\mu\cdot x)\phi(x)^n dx \\
    &= \tfrac{1}{2\pi}\int_{\mathbb{R}_+\times[0,2\pi)} \exp(i\mu_1r\cos\theta +i\mu_2 r\sin\theta)\phi(r)^n rdrd\theta/2\pi \\
    &= \tfrac{1}{2\pi}\int_{\mathbb{R}_+} \phi(r)^n \phi(|\mu|r) rdr
    \end{align*}
    Bon, maintenant il faut avoir le résultat en vue :
    \begin{align*}
    \mathbb{P}(S_n\in B(0,1))
    &= \int_0^1 \int_0^\infty \phi(\mu r)\phi^n(r) r\mu drd\mu\\
    &= \int_0^\infty \phi^n(r) \left\{ \int_0^1 \phi(\mu r)\mu rd\mu \right\} dr,
    \end{align*}
    par Fubini (avec justification et il y a un piège). Maintenant il faut avoir vécu au XIXè pour se rappeler que ce qui entre accolades peut s'écrire
    $$
    1/r \int_0^r \phi(t) tdt =-\phi'(r),
    $$
    ça vient de la définition de $J_0$ par une équa diff : $r^2\phi'' +r\phi'+r^2\phi=0$, i.e.
    $$
    (r\phi')'=-r\phi.
    $$
    Finalement (ouf!)
    $$
    \mathbb{P}(S_n\in B(0,1))=-\int \phi^n \phi'=\left[-\frac{\phi}{n+1}\right]_0^\infty =1/(n+1).
    $$

    Vous avez suivi (ça m'a épuisé moi) ?
    Bon courage,
    Lucas (scribe).
  • Ooooooooh comme c'est vilain les mauvaises langues pendant que je me tapais tout le LateX...

    Je précise à nouveau que je n'y suis pour rien dans cette preuve!
  • J'ai pas tout pigé, mais ça donne envie d'aller vivre au XIXème siècle.

    a+
  • tres joli solution!
    une solution tres similaire a ete proposee sur un autre forum par Fedja Nazarov - le point crucial etant comme ici de passer dans le domaine de Fourier. Le fait que la transformee de Fourier soit (presque) autoadjointe montre que la probabilite recherchee est aussi egale a: (a une constante multiplicative pres qui depend des conventions sur la transformee de Fourier):
    \[ \iint_{\R^2} \hat{A}^n(x) \hat{B}(x) \, dx\]
    ou $\hat{A}$ est la transformee de Fourier de la mesure uniforme sur le cercle unite, et $\hat{B}$ est la transformee de Fourier de la mesure uniforme sur le disque unite. Apres, on peut tout simplifier car tout est radiale, et comme la solution presentee plus haut, il y a un lien entre $\hat{A}$ et $\hat{B}$ qui simplifie tout (qui resulte de l'EDO de Bessel si on est savant, ou d'une integration par partie sinon).

    C'est au fond exactement la meme solution que celle precedente, traduite un petit peu differemment.
  • Very impressive ! Bravo à toi et à ton chef, et merci d'avoir pris la peine de taper tout ça. Après ton teasing sur Fourier j'avais essayé de calculer la TF de $X_1$ mais j'en étais resté à l'invariance par rotation sans reconnaître une fonction de Bessel (n'ayant pas vécu au XIXème siècle). Je t'avoue qu'il a deux ou trois passages ou j'ai décroché ce qui m'a empêché de repérer le "point magique" mais je relirai ça tranquillement ce soir au coin du feu.

    Pour le Fubini, il me semble si j'en crois Wikipedia qu'on a une majoration de la forme $|\varphi(r)| \leq A1_{[0,C]}(r)+\frac{B}{\sqrt{r}}1_{]C,+\infty[}(r)$ pour $A,B,C>0$, ce qui permet de majorer l'intégrale intérieure $$\int_0^\infty |\phi(\mu r)\phi^n(r)| r dr$$ en la découpant selon les trois intervalles $[0,C]$, $[C,C/\mu]$ et $[C/\mu,+\infty[$. On obtient au final une majoration en $D+E \mu^{n/2-2}$, qui multipliée par $\mu$ donne juste ce qu'il faut pour avoir l'intégrabilité en $0$ (grâce à $n>0$).
  • @alekk :

    > qui resulte de l'EDO de Bessel si on est savant,
    > ou d'une integration par partie sinon

    Oui, bien sûr le fait de reconnaître en $\phi$ la fonction de Bessel n'est pas indispensable, on peut juste essayer de voir si elle vérifie une EDO simple (on a bien envie vu qu'on a $\phi^n$ et qu'on veut l'intégrer pour avoir $1/(n+1)$).

    @egoroff :

    Ah bien vu ce découpage, l'argument de Nath. pour Fubini ne marchait que pour $n\geq 4$.
  • Messieurs,
    Je n'ai rien compris de cette résolution, mais ne vous en inquiétez pas. Le résultat à lui tout seul est déjà étonnant.
    Je suis très honoré d'avoir pu prendre part à cette discussion.:)
  • Un peu de bibliographie: ceci est un exercice du Spitzer 'Principles of random walks' (1964) Page 109. Pas clair qui a trouve ca, a mon avis le resultat final est du a Bochner (1949) qui a bien exploite les informations dans Watson 'A treatise on the theory of Bessel functions' (1922) Section 13.48 'the problem of random flights' ou sont resumes les idees de K. Pearson, Kluyver et Rayleigh=lord Kelvin.
  • Merci pour les références.
  • Bonsoir,

    J'ai continué à penser à cette marche aléatoire, avec l'espoir de résoudre ce problème avec mon (trop?) petit bagage théorique. Je n'arrive pas à dépasser le rang 3.

    Voici une illustration pour le rang 4:
    Chacun de ces 10 000 points est l' extrémité d'une somme de 4 vecteurs unitaires de directions aléatoires représentée à partir de l'origine:
    devant la somme $\vec {u_1}+\vec {u_2}+\vec {u_3}+\vec {u_4}$.
    Ça revient à faire une translation de la figure précédente:
    Le disque limité par le cercle bleu devrait donc contenir environ 1667 extrémités.

    Je n'arrive pas bien à le prouver, car la densité de proba suivant le rayon R se complique à chaque rang...14872
    14873
  • Et au fait, quelqu'un sait si en dimension supérieure la formule est aussi jolie?
  • Bonjour Lucas,

    J'essaye déjà de répondre à ce qui est à ma portée:
    Au rang inférieur, espace de dimension 1,
    Notre mobile ne peut faire qu'un pas en avant ou un pas en arrière. La réponse sera bien différente selon que l'on se pose la question du retour dans la boule ouverte de rayon 1 ou dans la boule fermée de rayon 1.

    Le retour dans la boule ouverte n'est possible que si le nombre de pas est pair: $n=2p$ la probabilité de ce retour est alors, corrige-moi si nécessaire:
    $\dfrac {C_{2p}^p}{2^{2p}}=\dfrac{(2p)!}{(p!)^22^{2p}}$



    Si le nombre de pas est impair $n=2p+1$, le retour n'est possible que sur le bord de la boule et sa probabilité est:
    $\dfrac {C_{2p+1}^p+C_{2p+1}^{p+1}}{2^{2p+1}}=\dfrac {C_{2p+2}^{p+1}}{2^{2p+1}}$

    Je ne vois pas de simplification vers un joli résultat.

    Au rang supérieurérieur,marche aléatoire dans un espace de dimension 3, je sais traiter le cas n= 2 et peut-être n=3 si le calcul de l'intégrale est à ma portée...

    n=2 Le mobile fait un premier pas dans une direction aléatoire puis un deuxième. On s'intéresse à la probabilité du retour dans la boule unitaire.
    Cette probabilité est égale à l'aire de la calotte sphérique de centre disons (1;0;0) et de rayon 1 contenue dans la boule unitaire de centre (0;0;0) rapportée à l'aire de la sphère toute entière
    On a donc: $P=\dfrac{2\pi*1^2*\frac {1}{2}}{4\pi}=\dfrac{1}{4}$

    Pour n=3, le calcul ne me semble pas trop horrible. Je m'y essayerai si tu penses que ça présente un intérêt..
  • En dimension 1, on est d'accord, pas de simplification possible. C'est bien pour ça que la formule me paraissait magique en dimension 2!

    En dimension 3 $n=2$, hé hé B-)-. J'avais même pas pensé à essayer. Ca rend optimiste ce 1/4... Bon, pour $n=3$ ça va pas être jojo mais ça se fait (j'ai la flemme très honnêtement).
  • Cher Lucas,
    Pour le calcul relatif à n=3, voici déjà une figure explicative:
    rapportée à l'aire de $S_M$ (ensemble des favorables / ensemble des possibles)

    Je pense le faire par intégration suivant x. Cette moyenne sera pondérée par les aires des tranches de $S_1$ d'épaisseur dx.

    Je ne suis pas sûr que ça donne un résultat simple, je calculerai plus tard si personne ne me devance, ce qui ne me fâcherait pas!

    A noter que le centre I de la base de la calotte est le milieu de [OM]

    Rappel: l'aire d'une tranche de sphère est égale à $2\pi R²h$ où h est son épaisseur et R le rayon de la sphère.

    L'organisation du calcul est-elle correcte?
    Edit: j'ai corrigé laire de la tranche qui était fausse dans mon message initial15366
  • Maintenant un peu de calcul:
    $OM^2=x^2+1-(1-x)^2=2x$ donc $OM= \sqrt {2x}$
    Alors la hauteur de la calotte jaune est $h=1-\dfrac {\sqrt{2x}}{2}$
    Donc l'aire de cette calotte est: $2\pi * 1^2*h=\pi (2-\sqrt{2x})$
    Si après deux pas, le mobile se trouve en M, la proba pour qu'au 3ème pas il finisse son parcours dans la boule $B_0$ est $\dfrac {\pi (2-\sqrt{2x})}{4\pi}=\dfrac{2-\sqrt{2x}}{4}$
    Maintenant, je calcule la moyenne de cette proba pondérée par les aires des tranches bleues :
    $$P=\dfrac {1}{4\pi}\int_0^2 \dfrac{2-\sqrt{2x}}{4} 2\pi dx = \dfrac {1}{4}\int_0^2 (1-\dfrac{\sqrt{2}}{2}\sqrt x ) dx $$
    $$P=\frac{1}{4} \left[x-\frac{\sqrt{2}}{3}x\sqrt x \right]_0^2 = \frac {1}{4}( 2-\frac{4}{3})$$
    Et, finalement: $P=\dfrac{1}{6}$
    
    Lucas ou quelqu'un voudrait-il reprendre ces calculs, ou peut-être faire une simulation pour vérifier?

    Le résultat est simple et me paraît plausible.
    Pour n=4, je serai bien incapable de faire le calcul...
  • Bonjour,
    2 remarques matinales:

    1) Dans l'étude du cas trivial de la boule de dimension 1, il y avait quand même une simplification possible:
    pour $n=2p+1$, on a: $C_{2p+1}^p=C_{2p+1}^{p+1}$ , d'où la simplification:
    $\dfrac {C_{2p+1}^p+C_{2p+1}^{p+1}}{2^{2p+1}}=\dfrac {C_{2p+1}^{p}}{2^{2p}}$
    Mais ça n'apporte pas grand chose de plus. On peut calculer les premiers termes de ces suites pour voir si une formule plus simple s'en dégage. Rien ne m'est sauté aux yeux.

    2) En dimension 3, pour $n=3$ , j'insiste sur le fait que je calcule une moyenne pondérée.
    Eh bien , présentement c'était inutile!
    Toutes les tranches de sphères d'épaisseur $dx$ (en bleu sur la figure) ont des aires égales.
    Je ne sais pas très bien si l'on doit ce théorème à Archimède ou à Lambert, en tous cas ce dernier a utilisé cette propriété en cartographie pour les projections équivalentes qui portent son nom...
  • C'est du a Archimede, comme en 1955 et c'etait demontre dans mon livre de 1ere. Archimede etait si fier de la decouverte qu'il a fait dessiner un cylindre circonscrit a une sphere sur sa tombe, ce qui a permit a Ciceron de localiser celle ci 130 ans plus tard. Amicalement.
  • Salut Jacquot,

    J'ai lancé un million de simulations, en trois dimensions, pour la somme de $n=2,3$ ou $4$ points. J'obtiens les intervalles de confiances à $95\%$ suivants : $$n=2 \quad ; \quad I_n=\left[ \, 0.24907 \, ; \, 0.25076 \, \right]$$ $$n=3 \quad ; \quad I_n=\left[ \, 0.16545 \, ; \, 0.16691 \, \right]$$ $$n=4 \quad ; \quad I_n=\left[ \, 0.12055 \, ; \, 0.11927 \, \right]$$ Il me semble que ça confirme expérimentalement tes calculs aboutissant à $1/4$ et $1/6$ pour $n=2$ et $3$, et comme ça si tu trouves un résultat pour $n=4$ tu pourras comparer avec le $0,12$ suggéré par les simulations.
  • (tu)Merci egoroffski,

    Pour n=4, je ne pense pas être capable de mener à bien les calculs: je n'ai déjà pas su le faire pour des déplacements dans le plan, du fait qu'il faut tenir compte de la densité de proba qui n'est pas uniforme après 3 déplacements.

    Pas sûr, cependant que le problème dans l'espace soit plus difficile que dans le plan:
    pour n=3, le calcul intégral dans l'espace s'est avéré plus simple que dans le plan grâce à la propriété d'Archimède citée plus haut.

    Ah, si ta simulation avait donné une proba de 0,125 au lieu de 0,12 c'eût été plus encourageant!;)

    Je pense que Lucas voudrait savoir s'il existe, dans l'espace, un traitement général puissant du type de celui qui a été présenté dans le plan [size=x-small](et que je ne suis pas près de comprendre!)[/size]
  • > Je pense que Lucas voudrait savoir s'il existe, dans l'espace, un traitement général
    > puissant du type de celui qui a été présenté dans le plan

    Exactement, disons que c'était la question que je me posais avant qu'une nouvelle question occupe mon esprit : "quand est-ce que cette $£@&amp;$£à de cendre va retomber histoire que je puisse prendre mon avion pour Boston".

    Sinon jacquot, il faut pas beaucoup savoir de probas pour la preuve du dessus. Juste que la transformée de Fourier caractérise la loi d'une variable aléatoire. Après c'est du calcul (que je n'aurais pas mené moi-même!).
  • Sinon tu peux prendre un taxi jusqu'à Lisbonne et décoller de là.
  • Lucas : c'est juste pour voir Peter Gacs en vrai ?
  • Tiens tiens... qui se cache derrière ce Grotte de Nez (très joli prénom), il y a peu de gens qui savent que je m'intéresse à Gacs...

    Pour répondre à la question, j'y allais pour le loisir, mais là c'est définitivement tombé à l'eau je crois, ou dans la cendre plutôt.
  • Bonjour à tous,

    A votre avis existe-t-il un lien entre le résultat de ce fil passionnant

    et la propriété géométrique suivante :

    si les vecteurs unitaires v_i sont en nombre impair et s'ils sont situés

    sur un même demi-cercle ouvert, alors la norme de leur somme

    dépasse 1 ?

    Cordialement,
  • Je voudrais savoir aussi comment mémoriser et/ou imprimer

    les 47 messages de ce fil ?

    Merci d'avance pour vos réponses.
  • Bon, Lucas,
    Alors je te propose un peu de calcul pour meubler ton week end.

    Comme je l'ai évoqué plus haut, les calculs en dimension 3 me semblent plus faciles qu'en dimension 2, notamment du fait de cette propriété d'Archimède, le calcul de l'aire d'une calotte sphérique est plus facile que le calcul de la longueur d'un arc de cercle.

    Il reste alors un peu d'espoir pour établir un processus qui nous permettrait de calculer $P(|S_n|\leq 1)$ par récurrence.
    Poue ce faire, je cherche une fonction de répartition de notre proba et j'essayerai de calculer $P(|S_n|\leq R)$ où le rayon R de la boule de centre O sera variable
    n=1, il est sûr que $x=1$
    Pour n=2, on aura $P(|S_2|\leq R]=\dfrac{R^2}{4}$ puisque $x=1$
    Avec le cas particulier: $P(|S_2|\leq 1]=\dfrac{1}{4}$, pour rappel.
    Pour n=3, je calcule d'abord $P(x\leq |S_2|\leq x+\Delta x)= \dfrac{(x+\Delta x)^2}{4}-\dfrac{x^2}{4}$ qui est équivalent à $\dfrac{x}{2}\Delta x$ pour $\Delta x$ petit
    $$P(|S_3|\leq R)=\int_{|R-1|}^{R+1}\frac {R^2-(x-1)^2}{4x}*\frac{x}{2}dx= \frac {1}{8}\left [(R^2-1)x+x^2-\frac{x^3}{3}\right]_{|R-1|}^{R+1}$$
    qui se calcule bien pour le cas particulier $R=1$, obtenant $\dfrac{1}{6}$ qui confirme le résultat obtenu précédemment.

    Je n'ai pas le temps de continuer ...
    @ suivre.15406
  • Bonjour.
    Si $X_1,\ldots, X_n$ sont iid et r\'epartis uniform\'ement sur la sph\`ere unit\'e de $\mathbb{R}^3$ alors -comme d\'ej\`a dit il y a deux mois ici- alors pour $n\geq2$ le nombre $p_n=\Pr(\|X_1+\cdots+X_n\|<1)$ est donn\'e dans Watson '\textit{A Treatise on the Theory of Bessel Functions}' section 13.48. En particularisant la page 421 on obtient
    $p_n=\frac{1}{\pi}(I_{n+1}-J_n)$ avec
    $$I_n=\int_{-\infty}^{\infty}\left(\frac{\sin t}{t}\right)^ndt,\ J_n=\int_{-\infty}^{\infty}\left(\frac{\sin t}{t}\right)^n\cos t \, dt$$
    (Dans Watson il faut aller chercher les expressions des fonctions de Bessel $J_{1/2}$ et $J_{3/2}$ page 54). Posons $f_n(x)=\frac{1}{2\pi}\int_{-\infty}^{\infty}\left(\frac{\sin t}{t}\right)^ne^{-ixt}dt:$
    c'est la densit\'e de $Y_1+\cdots+Y_n$ lorsque les $Y_j$ sont iid et uniformes sur $[-1,1]$ (en effet la transform\'ee de Fourier de $X_j$ est $\frac{\sin t}{t}$ et pour $n\geq 2$ la formule d'inversion est applicable). En particulier $f_n$ est continue. Donc finalement $p_n=2f_{n+1}(0)-f_n(-1)-f_n(1)=2(f_{n+1}(0)-f_n(1))$ puisque $f_n$ est paire.
    Il se trouve que $f_n$ est bien connue (voir A. Renyi, \textit{Probability Theory} page 192): pour $|x|<n$ on a si $[y]$ signifie partie enti\`ere de $y>0:$
    $$f_n(x)=\frac{1}{(n-1)!2^n}\sum_{k=0}^{[(n+x)/2]}(-1)^kC^k_n(x+n-2k)^{n-1}$$
    Donc $$p_n=\frac{1}{n!2^n}\sum_{k=0}^{[(n+1)/2]}(-1)^{k+1}C^k_{n+1}(n-1+\frac{2k}{n+1})(n+1-2k)^{n-1}.$$
    avec par exemple $p_2=1/4,p_3=1/6, p_4=23/192=0,11971666....$ ce qui confirme tous les calculs num\'eriques du fil. La th\'eorie pr\'evoit $\sum p_n<\infty$ mais peut on avoir une \'evaluation asymptotique de $p_n?$ Probablement $\ell=\lim n^{3/2}p_n$ existe et est finie, mais quel est $\ell?$
  • Merci Gérard,

    J'étais en train de taper la suite de mes calculs laborieux: je m'attaquais au cas n=4 sans arriver à trouver ce qui clochait dans mes calculs (difficultés dues à la valeur absolue sur l'une des bornes d'intégration)
    quand tu m'as devancé.
    Ainsi donc tu réponds à la question de Lucas, et je peux jeter mes brouillons à la corbeille.;)
Connectez-vous ou Inscrivez-vous pour répondre.